You are on page 1of 15

ITESM Campus Monterrey

Mathematical Physical Modelling F4005


HW6: Multivariable calculus part I
Due Date: April 1-2019, 23:59 hrs.
Professor : Ph.D Daniel López Aguayo

Full names of team members: Valeria Martı́nez Villarreal, Karla Liliana Tarango Bustamante, Aida
Taravat Fard, Jesús Martı́n De Los Santos Ramı́rez, Daniel Hugo Solano Teran, Chinmay Pramodkumar
Tiwari.
Instructions: Write neatly on each page of your homework and send it in pdf format to dlopez.aguayo@tec.m
Typed solutions in LATEX(only) will be given extra credit; no late homework will be accepted.

1 Find ∂f , ∂f if
∂x ∂y
(a) f (x, y) = xy x
∂f
∂x
= y x + (lny)y x (x)
∂f
∂y
= y x−1 x2

(b)f (x, y) = exy


∂f
∂x
= yexy
∂f
∂y
= xexy

(c) f (x, y) = xcos( πx )cos(πy)


x
∂f xsin π )cos(πy)
∂x
= cosπycos πx − π

∂f
∂y
= −πxsin(πy)cos( πx )

1
(d) f (x, y) = (x2 + y 2 )ln(x2 + y 2 )
∂f
∂x
= 2x.ln(x2 + y 2 ) + 2x
∂f
∂y
= 2y.ln(x2 + y 2 ) + 2y

∂z ∂z
2 Evaluate the partial derivatives ∂x , ∂y for the given function z = f (x, y) at the indicated points.
p
(a) f (x, y) = a2 − x2 − y 2 ; (0, 0), ( a2 . a2 )
∂z 1
∂x
= (a2 − x2 − y 2 ) 2
1 −1
= 12 (a2 − x2 − y 2 ) 2 −1 (−2x) → 21 (a2 − x2 − y 2 ) 2 (−2x); x = 0, y = 0
−1
= 12 (a2 − 0 − 0) 2 (2(0)) = 0

1 2 a2 a2 −1 −2
2
(a − 2
− 2
) 2 (−2( a2 )) = 2

(b) f (x, y) = ln 1 + xy; (1, 2), (0, 0)
1
∂f (1+xy)− 2 y
∂x
= 1 y= 2(1+xy)
2(1−xy) 2

∂f (1,2) 1
∂x
= 3

1
∂f (1+xy)− 2 x
∂y
= 1 x= 2(1+xy)
2(1−xy) 2

∂f (1,2) 1
∂y
= 6

2
(c) f (x, y) = eax cos(bx + y); ( 2π
b
, 0)
∂f 2πa
∂x
= aeax (cos(bx + y)) + (−sin(bx + y)(b)(eax ) = ae b

∂f
∂y
= −eax sin bx + y

∂f ( 2π ,0)
∂y
b
=0

∂2f ∂2f
p
3 Let f(x, y) = ln x2 + y 2 . Prove that f satisfies the so-called Laplace equation: ∂x2
+ ∂y 2
= 0.
Solution:
∂f x
a) ∂x
= x2 +y 2
∂2f ∂ x2 +y 2 −2x2 y 2 −x2
∂x2
= ( x )
∂x x2 +y 2
= (x2 +y 2 )2
= (x2 +y 2 )2
∂f y
b) ∂y
= x2 +y 2
∂2f x2 +y 2 −2y 2 x2 −y 2
∂y 2
= ∂
( y )
∂y x2 +y 2
= (x2 +y 2 )2
= (x2 +y 2 )2
∂2f ∂2f y 2 −x2 x2 −y 2
a)&b) =⇒ ∂x2
+ ∂y 2
= (x2 +y 2 )2
+ (x2 +y 2 )2
=0

4 Sketch the curves in R2 that are the images of the paths in the given exercises.
Then verify your answers with Mathematica. Hint: use the P arametricP lot command.
(a) x = sint, y = 4cost, 0 ≤ t ≤ 2π.
Solution:
(
x(t) = sint y2
y(t) =⇒ x2 + 16
= 1,=⇒ As we know the equation of an ellipse with the center of (0,0) is
4
= cost
x2 y2
a2
+ b2
= 1. It’s the equation of a vertical ellipse with a=1 and b=4 and the center of (0,0).

(b) x = 2sint, y = 4cost, where 0 ≤ t ≤ 2π. Solution:


(
x(t)
= sint 2 2
2
y(t) =⇒ x4 + y16 = 1, =⇒ As we know the equation of an ellipse with the center of (0,0) is
4
= cost

3
x2 y2
a2
+ b2
= 1. It’s the equation of a vertical ellipse with the center of (0,0) and a=2 and b=4.

5 Consider the circle C of radius 2, centered at the origin. Find a parametrization for C inducing a
counterclockwise orientation and starting at (2, 0). Also, verify your answer with Mathematica.
x=rcost −→ x=2cost
y=rsint −→ y=2sint
c(t)=(2cost,2sint) where 0 ≤ t ≤ 2π

6 In the following exercises, find the velocity vector of the given path.
(a) c(t) = 6ti + 3t2 j + t3 k
c’(t)=v(t)=6i+6tj+3t2 k
3
(b) c(t) = (sin3t)i + (cos3t)j + 2t 2 k
1
c’(t)=v(t)=(3cos3t)i-(3sin3t)j+ 32 (2t) 2 k
1
v(t)=(3cos3t)i-(3sin3t)j+3t 2 k
v(t)=(3cos3t)i-(3sin3t)j+ √3t

7 Give a parametrization for each of the following curves:


(a) The line in R3 passing through (1, 2, 3) and (2, 0, 7). Hint: review the parametric equation of a
line in R3 .
A=< 1, 2, 3 >
B=< −2, 0, 7 >
−→
AB=B-A=< −2 − 1, 0 − 2, 7 − 3 >=< −3, −2, 4 >
Then, we do the parametric equation of a line −→ c(t) =< 1, 2, 3 > + < −3, −2, 4 > t
c(t)= < 1, 2, 3 > + < −3t, −2t, 4t >
c(t) =< 1 − 3t, 2 − 2t, 3 + 4t >
(b) The arc of the parabola in R2 given by y = x2 and that goes from (1, 1) to (3, 9).
x=t
y=t2
c(t)=(t,t2 ) where 1 ≤ t ≤ 3
(c) The square with vertices (0, 0), (0, 1), (1, 1) and (1, 0) (Break it up into 4 line segments.) Also,
use the Show command to plot (in the same plane) the four parametrizations and verify that you get

4
the square.
1st vertex: (0,0)
2nd vertex: (0,1)
3rd vertex: (1,1)
4th vertex: (1,0)


v1 =< 0, 1 >-< 0, 0 >=< 0, 1 >


v2 =< 1, 1 >-< 0, 1 >=< 1, 0 >


v3 =< 1, 0 >-< 1, 1 >=< 0, −1 >


v1 =< 0, 0 >-< 1, 0 >=< −1, 0 >
Parametrizations:
1)< x, y >=< 0, 0 >+t< 0, 1 >
2)< x, y >=< 0, 1 >+t< 1, 0 >
3)< x, y >=< 1, 1 >+t< 0, −1 >
4)< x, y >=< 1, 0 >+t < −1, 0 >
c1 (t)=< 0, t >
c2 (t)=< t, 1 >
c3 (t)=< 1, 1 − t >
c4 (t)=< 1 − t, 0 >
2 2
(d) The ellipse given by x9 + y25 = 1.
cos2 x+sin2 y=1
x
a2
+ by2 =1
a=3
b=5
x=rcost
y=rsint
c(t)=< 3cost, 5sint > where 0 ≤ t ≤ 2π

2 3
8 Show (by hand)
√ that the directional derivative of f(x, y, z) = z x + y at (1, 1, 2) in the direction
i 2j
of √5 + √5 is 2 5.

D u#» f (p) = ∇f (p) · #»


u
∇f=< ∂f , ∂f ∂f
,
∂x ∂y ∂z
>
∂f 2
∂x
=z
∂f
∂y
=3y 2
∂f
∂z
=2z
∇f=< z 2 , 3y 2 , 2z >
∇f(p)=< 4, 3, 4 >
∇f (p) · #» u =< 4, 3, 4 > · < √1 , √2 , 0
5 5
>

D u#» f (p) = √45 + √65



D u#» f (p) = 2 5
9 Compute with Mathematica the directional derivatives of the following functions at the indicated
points in the direction of the given vectors:
(a)f (x, y) = x + 2xy − 3y 2 , (x0 , y0 ) = (1, 2), v = 35 i + 45 j

p 2i√+j
(b)f (x, y) = ln( x2 + y 2 ), (x0 , y0 ) = (1, 0), v = 5

5
(c)f (x, y) = ex cos (πy), (x0 , y0 ) = (0, −1), v = − √15 i + √2 j
5

10 Let f : R3 →
− R be a function, x0 a point in R3 and let v be a unit vector in R3 . Prove that the
maximum value of the directional derivative of f at the point x0 along v is ||∇f (x0 )||. What is the
minimum value?
D v#» f (x0 ) = ∇f (x0 ) · #»
v
∇f (x ) · #»v = ||∇f (x )|| · || #»
0 0 v || cos θ
Knowing that || #»
v || = 1, and the maximum value of cos θ = 1.
∴ We can infer that the maximum value of the directional derivative of f at the point x0 along v will
be:
D v#» f (x0 ) = ||∇f (x0 )||
The minimum value will be when cos θ = −1.

∴ D v#» f (x0 ) = −||∇f (x0 )||

11 Use the P lot3D command to graph the surface f (x, y) = 100 − 2x2 − 3y 2 .

6
(a) What are the xy coordinates of the highest point on the graph of f ?
∂f
= −4x −→ −4x = 0 ∴ x = 0
∂x
∂f
= −6y −→ −6x = 0 ∴ y = 0
∂y
∂ 2f
= −4
∂x2
∂ 2f
= −6
∂y 2
Hessian Hessian Matrix:
−4 0
H = = 24
0 −6
H>0→
− There is a Max. or Min. value
∂2f
∴ As the ∂x2
< 0. The values of ∂f
∂x
= 0 and ∂f
∂y
= 0 indicates the points of x and y in the function f in
order to get its maximum value.
∴ f (x, y)max = 100.
(b) Show that the gradient of f is the zero vector at the point found in (a). Do you recall (from
Mathematics III) why this is the case?

∇f = h−4x, −6yi

∇f (0, 0) = h0, 0i
The gradient points in the direction of greatest increase of a function. Therefore is zero at a local
maximum or local minimum (because there is no single direction of increase).
12 Let r = xi + yj + zk and r = ||r ||. Prove that ∇( 1r ) = − rr3 .

r = hx, y, zi
p
||r || = r = x2 + y 2 + z 2
1 1
=p
r x + y2 + z2
2

∂( 1r ) x
=− 3
∂x (x2 + y 2 + z 2 ) 2
∂( 1r ) y
=− 3
∂y (x2 + y 2 + z 2 ) 2
∂( 1r ) z
=− 3
∂z (x2 + y 2 + z 2 ) 2
1 x y z
∇( ) = h− 3 ,− 3 ,− 3 i (1)
r (x2 + y 2 + z 2 ) 2 (x2 + y 2 + z 2 ) 2 (x2 + y 2 + z 2 ) 2
p 3
r3 = ( x2 + y 2 + z 2 )3 = (x2 + y 2 + z 2 ) 2

r x y z
− 3
= h− 3 ,− 3 ,− 3 i (2)
r (x2 + y 2 + z 2 ) 2 (x2 + y 2 + z 2 ) 2 (x2 + y 2 + z 2 ) 2
∴ From equation 1 and 2 we can say that ∇( 1r ) = − rr3 . 13 In electrostatics, the force P of attrac-
tion between two particles of opposite charge is given by P = k(r/||r3||) (Coulomb’s law), where k is a
constant and r = xi + yj + zk. Prove that P is the gradient of f = −k/||r||.

7
Hint: this is immediate if you use the previous exercise!
Solution: Using result from previous example when r = ||r||
(∇(1/r) = −r/r3 ) —– Result (1) from previous example
Given Function, f = −k/||r||
∇(f ) = ∇(−k/||r||) = −k × ∇(1/||r||)
∇(f ) = −k × ∇(1/r) —– from result (1)
∇(f ) = −k × (−r/r3 )
∇(f ) = −k × (−r/||r3 ||) —– given in the result for previous question (r = ||r||)
∇(f ) = k × (r/||r3 ||) = P → (which is equivalent to value of P given in question)
Hence, proved.

14 A bug finds itself in a toxic environment. The toxicity level is given by T (x, y) = 2x2 − 4y 2 . The
bug is at (−1, 2).
(a) Prove√that in order
√ to lower the toxicity the fastest, the bug must move in the direction of the unit
vector i/ 17 + 4j/ 17 .
(b) Now go to https://www.geogebra.org/m/sWsGNs86, then plot the surface T and the gradient of T
at the point (-1, 2). Can you see why (geometrically) the answer from the above item makes sense?
Solution: ∇T = h4x, −8yi , is the given vector in the question
∇T(−1,2) = h−4, −16i
√ √
∇T(−1,2) · v̂ = h−4, −16i · h1/ 17, 4/ 17i
√ √
∇T(−1,2) · v̂ = (−4/ 17 − 64/ 17)
√ √
∇T(−1,2) · v̂ = −68/ 17 = −4/ 17
Negative value of the directional derivative suggests the path with towards the minimum toxicity (If
the value would have been positive it would lead to the maximum toxicity value).

In the above figure it can be clearly seen that the√ unit vector
√ (in yellow) points in the direction of
maximum toxicity. Unit vector in this case is h−1/ 17, −4/ 17i but if we multiply the unit vector by
(-1) we get a vector in √
opposite√direction which will point in the direction of minimum toxicity and the
unit vector will be h1/ 17, 4/ 17i

2 2
15 Let h(x, y) = 2e−x + e−3y denote the height on a mountain at position (x,y). In what direction
from (1, 0) should one begin walking in order to climb the fastest ?
Solution: Gradient of a function always direct towards the maximum value of a function hence we will
find a gradient for the following function
2 2
∇h(x,y) = h−4x · e−x , −6y · e−3y i
∇h(1,0) = h−4/e, 0i

8
Figure 1: Gradient of function at point (-1,2), (From Geogebra)

Thus for climbing faster on the mountain one should start moving from (1,0) towards the point (-4/e,0)

16 Let C be a circle of radius r. Using a parametrization of C and the Arc length formula given in
class, prove by hand that the perimeter of C is 2πr
Rt
Solution: Arc length is given by= t01 ||c0 (t)||dt
c(t) = (r cos t, r sin t)
c : [0, 2π) → R2
c0 (t) = (−r sin t, r cos t)

||c0 (t)|| = r2 sin2 t + r2 cos2 t

||c0 (t)|| = r2 — (as we are aware about identity sin2 t + cos2 t = 1)
||c0 (t)|| = r
Integrating for the above function for limits [0, 2π), we get
R2
Arc length = 0 rdt
Arc length = [rt]2π
0

Arc length = 2πr


Hence, Proved

17 Use the ListPointPlot3D and the Table commands to draw the curve inR3 given by r(t) =
(cos t, sin t, t2 ),where t ∈ [0, π]. Once done this, use Mathematica to prove that that its length is,
approximately, 10.6281. Hint : To get a nice graph add a step size of 0.01 (this is an optional parame-
ter)
Solution: Hereby attaching the screenshot of Mathematica calculation

Figure 2 indicates ListPointPlot3D function


Figure 3 indicates about calculation of Arc length

9
Figure 2: Plotting using ListPointPlot 3D function

Arc length comes out to be the same as indicated in the question


18 Give a parametrization of a circle with center (h , k) and radius r.

Solution:
2 2
c(t)= (x(t), y(t)) ,we know that the formula of a circle with center (h, k) and radius r is (x−h)
r2
+ (y−k)
r2
=1
x−h y−k
then r = Sin(t) and r = Cos(t). So x(t)= rCos(t)+h, y(t)=rSin(t)+h =⇒ c(t)= (rCos(t)+h,
rSin(t)+h) , t∈[0,2π].

19 Sketch the curve in R2 given in parametric form by x(t) = 2sint, y(t) = 4cost, where t∈[0, 2π).

Solution:

10
Figure 3: Arc length calculation

20 Prove, by hand, that the arc length of the curve


√ √ 2
c(t) = (ln( t), 3.t, 3t2 )f ort ∈ [1, 2].
9+ln 2
is 2
. Then, use Mathematica to verify this.

Solution:
R2 0
1
kc (t)kdt =?
0
√ 0
q q
2 4 +1 6t2 +1 6t2 +1
c (t) = ( 2t , 3, 3t) =⇒ kc (t)k = 4t2 + 3 + 9t = 12t +36t
1 1 2
4t2
=| 2t
| −→ t ∈ [1, 2] =⇒| 2t
| =
6t2 +1
2t
R 2 6t2 +1 R2
1 2t
dt = 1
3t + 2t−1 dt = 32 t2 + 12 ln t, t ∈ [1, 2] = (6 + 21 ln2 − 32 ) = 9+ln 2
2

21
(a) Compute, by hand, the arc length of the curvec(t) = (sin2 t, cos2 t)where t ∈ [0, 2π).
Solution:
R 2π 0
kc (t)kdt =?
0 √
c (t) = (2Cos(t)Sin(t), −2Cos(t)Sin(t)) =⇒ kc0 (t)k = 4Cos2 Sin2 t + 4Cos2 Sin2 t =
0

11
p √ R 2π √ 2t=u.2dt=du→dt= du2
4Sin2 (2t) = | 2Sin(2t)| =⇒ 0 | 2Sin(2t)|dt =−−−−−−−−−−−−→=
√ R
2 4π
√ R
4 2 π
√ √
2 0
|Sin(u)|du = 2 0
Sin(u)du = −2 2Cos(u), t ∈ [0, π] = 4 2

(b) Compute the above item using Mathematica.


Solution:

(c) Compute (again) by hand, the arc length of the given curve by first finding a familiar representation
of c.
Solution:
(
X = Sint
Y = Cos2 t
( + Y = 1 =⇒ Y = 1 − X
X
0 ≤ Sin2 t ≤ 1 =⇒ 0 ≤ X ≤ 1
0 ≤ COs2 t ≤ 1 =⇒ 0 ≤ Y ≤ 1

( ( ( (
t = 0 −→ (0, 1) t = π/2 −→ (1, 0) t = π −→ (0, 1) t = 3π/2 −→ (1, 0)
t = π/2 −→ (1, 0) t = π −→ (0, 1) t = 3π/2 −→ (1, 0) t = 2π −→ (0, 1)

It means that the equation passed this line for four times −→ the solution is equal to 4× 2 21
(a) Compute, by hand, the arc length of the curvec(t) = (sin2 t, cos2 t)where t ∈ [0, 2π).
Solution:
R 2π 0
kc (t)kdt =?
0 √
c (t) = (2Cos(t)Sin(t), −2Cos(t)Sin(t)) =⇒ kc0 (t)k = 4Cos2 Sin2 t + 4Cos2 Sin2 t =
0
p √ R 2π √ 2t=u.2dt=du→dt= du
2
4Sin2 (2t) = | 2Sin(2t)| =⇒ 0 | 2Sin(2t)|dt =−−−−−−−−−−−−→=
√ R
2 4π
√ R
4 2 π
√ √
2 0
|Sin(u)|du = 2 0
Sin(u)du = −2 2Cos(u), t ∈ [0, π] = 4 2

(b) Compute the above item using Mathematica.


Solution:

12
(c) Compute (again) by hand, the arc length of the given curve by first finding a familiar representation
of c.
Solution:
(
X = Sint
Y = Cos2 t
( + Y = 1 =⇒ Y = 1 − X
X
0 ≤ Sin2 t ≤ 1 =⇒ 0 ≤ X ≤ 1
0 ≤ cOs2 t ≤ 1 =⇒ 0 ≤ Y ≤ 1

( ( ( (
t = 0 −→ (0, 1) t = π/2 −→ (1, 0) t = π −→ (0, 1) t = 3π/2 −→ (1, 0)
t = π/2 −→ (1, 0) t = π −→ (0, 1) t = 3π/2 −→ (1, 0) t = 2π −→ (0, 1)

It means that the equation passed this line for four times −→ the solution is equal to 4× 2 22 The tem-
2 2 2
perature at any point in a solid metal ball centered at the origin is given by T (x, y, z) = 100e−x −y −z .
(a) Where is the ball hottest?
If we compute the partial derivatives and equal them to 0:
2 −y 2 −z 2
∂T
∂x
= −200xe−x =0
2 −y 2 −z 2
∂T
∂y
= −200ye−x =0
2 −y 2 −z 2
∂T
∂z
= −200ze−x =0
From the latter x = 0, y = 0 & z = 0
Now we compute the second derivatives:
∂2T 2 −y 2 −z 2
∂x2
= −200e−x (1 − 2x2 )
∂2T 2 −y 2 −z 2
∂y 2
= −200e−x (1 − 2y 2 )
∂2T 2 −y 2 −z 2
∂z 2
= −200e−x (1 − 2z 2 )
From this we can see that the possible results when evaluating the equations at any point are always
less than 0, suggesting that the maximum is at the point (0, 0, 0).
2 −y 2 −z 2
Another way to see this is arranging 100e−x as:
100
ex2 +y2 +z2

13
From this the max value will be 100 at x = 0, y = 0 & z = 0
(b) Where is the ball the coolest? Does this question make sense? Explain mathematically.
Now regarding to the minimum, there is no a specific point where the function obtains a minimum
value. What we can do is to calculate the limit when (x, y, z) −→ ∞
2 −y 2 −z 2
limx,y,z→∞ 100e−x =0
”Getting” a minimum value at x = ∞, y = ∞ & z = ∞
(c) Also, show that at any point (x, y, z) on the ball, the direction of greatest increase in temperature
is a vector pointing towards the origin.
2 −y 2 −z 2 2 −y 2 −z 2 2 −y 2 −z 2
∇f (x, y, z) = h−200xe−x , −200ye−x , −200ye−x i
−x2 −y 2 −z 2
∇f (x, y, z) = −200e hx, y, zi
−x2 −y 2 −z 2
The factor −200e says that no matter the direction of the vector it is always going to be
”redirected” to the origin

23 The height of a mountain can be modeled by h(x, y) = 3000 − 2x2 − y 2 , where the x-axis points
north, the y-axis points east and all distances are measured in meters. Suppose that a mountain climber
is at point (30,-20,800).
(i) Make a plot of h in Mathematica.

(ii) Assuming the climber moves in the southwest direction, will he (or she) ascend or descend?
By looking at the plot we can easily say that the person will ascend.
And using the directional derivative: ∇h(x, y) = h−4x, −2yi
∇h(30, −20) = h−120, 40i
Now we need a unit vector v. Lets take it from the southwest direction.
a = h−1, −1, 0i
v = a/|a| = h− √12 , − √12 , 0i

h−120, 40, 0i · h− √12 , − √12 , 0i = 40 2
As the result is positive we can say that the direction Southwest is towards the highest value.

24 The area of a rectangle of base b and height h is bh. Prove this formula using a line integral and

14
an appropriate curve C.
R
C
hds where h represents a constant value
c(t) = h0, 0i + th1, 0i = ht, 0i
||c0 || = 1
Rb Rb
0
h||c0 ||dt = h 0 1dt = ht|b0 = hb
25 Evaluate the following line integrals.
R
(a) C yds, where C is given in parametric form by (t2 , t), t ∈ [0, 2].

||c0 (t)|| = 4t2 − 1
R2 √
0
t 4t2 − 1dt
u = 4t2 − 1, du = 8tdt
1 17 12 1 32 17
− 1) ∼
R 1 3
8 1
u du = 12 u |1 = 12
(17 2 = 5.757
R
(b) C
xy 4 ds, where C is the right-half of the circle x2 + y 2 = 2
√ √
||c(t)|| = ( 2 sin t, 2 cos t)
√ √
||c0 (t)|| = 2 cos t2 + 2 sin t2 = 2
Rπ√ 4
√ Rπ
0
2(sin t)4(cos t) 2dt = 8 0
sin t(cos t)4 dt
u = cos t, du = − sin t
Rπ 5
−8 0 u4 du = −8( u5 |π0 )
Going back to t.
5
−8( cos5 t |π0 ) = − 85 (cos π − cos 0) = − 85 (−1 − 1) = − 85 (−2) = 16
5

(c) Prove that C xyds = 5 6 2 where C consists of the line segments from (0,0) to (1,0) and from (1,0)
R

to (2,1). Hint: Write C as two distinct parametrized curves and split into two integrals.
c(t1 ) = h0, 0i + th1, 0i = ht, 0i
||c0 (t1 )|| = 1
R1
0
t(0)(1)dt = 0
c(t2 ) = h1, 0i + th1, 1i = h1 + t, ti

||c0 (t1 )|| = 2
R1 √ √ R1 R1
0
(1 + t)(t) 2dt = 2( 0 t2 dt + 0 tdt) =
√ t3 t2 1 √ 1 1 √ √
5 2
2( 3 + 2 )|0 = 2[( 3 + 2 ) − (0)] = 2( 56 ) = 6
R
(d) C x4 yds, where C is the curve y = x3 from (1,1) to (2,8).
c(t) = (t, t3 )
p √
||c0 (t)|| = 12 + (3t2 )2 = 1 + 9t4
R 2 4 3√
1
t t 1 + 9t4 dt
u = 1 + 9t4 , du = 36t3 , t4 = u−1
9
1
R 145 u−1 1 1
R 145 3 1
36 10
( 9 )u 2 du = 324 10 (u 2 − u 2 ) =
1 2 52
− 23 (145) 2 ] − [ 25 (10) 2 − 23 (10) 2 ]) ∼
3 5 3 5 3
( u
324 5
− 23 u 2 )|145
10 =
1
([ 2 (145) 2
324 5 = 308.643

15

You might also like